1
$\begingroup$

Consider a $\textbf{flat}$ surjective map $f: X \rightarrow \mathbb{A}^1$. The general fibers $F_{\epsilon}$ are canonically isomorphic, and the special fiber $F_0$ above $0 \in \mathbb{A}^1$ is not isomorphic to the general fibers.

Given a closed subscheme $B \subset X$, we define its special fiber limit $\widetilde{B}$ to be the intersection of the special fiber $F_0$ with the closure of $B \times \mathbb{A}^1 \backslash \{ 0 \}$ in $X$, $F_0 \cap \overline{B \times \mathbb{A}^1 \backslash \{ 0 \}}$.

Let $B_1$ and $B_2$ be two closed subschemes of $X$. When does the limit of their intersection equal to the intersection of their limits, i.e. $\widetilde{B_1} \cap \widetilde{B_2} = \widetilde{B_1 \cap B_2}$? Is it enough for the dimension of $\widetilde{B_1} \cap \widetilde{B_2}$ to be equal to the dimension of $B_1 \cap B_2$? Any relevant comments and references are welcome. Thanks!

$\endgroup$

1 Answer 1

4
$\begingroup$

Obviously $\widetilde{B_1} \cap \widetilde{B_2} \subseteq \widetilde{B_1\cap B_2}$. I'll discuss a sufficient condition for the reverse.

If $B_1\cap B_2$ is equidimensional, then so is $\widetilde{B_1\cap B_2}$. It seems like you know that its dimension is that of $\widetilde{B_1}\cap \widetilde{B_2}$. By any chance is $\widetilde{B_1}\cap \widetilde{B_2}$ reduced, and the family projective? Then if $\deg(B_1\cap B_2) = \deg(\widetilde{B_1}\cap \widetilde{B_2})$ (w.r.t. the projective embedding), your $\widetilde{B_1\cap B_2}$ has to be all of $\widetilde{B_1}\cap \widetilde{B_2}$.

$\endgroup$
2
  • $\begingroup$ Thanks! This sounds very nice. Why is it true? Is the proof/reasoning written somewhere? In my case, it is hard for me to figure out the degree of the projective embedding, but maybe I can find some other equivalent conditions... $\endgroup$
    – Qiao
    Mar 7, 2016 at 4:44
  • $\begingroup$ By flatness, $\deg(B_1\cap B_2) = \deg(\widetilde{B_1\cap B_2)}$. Then use Lemma 1.7.5 of my paper with Ezra Miller on Schubert polynomials. arxiv.org/pdf/math/0110058.pdf $\endgroup$ Mar 7, 2016 at 12:34

Your Answer

By clicking “Post Your Answer”, you agree to our terms of service and acknowledge you have read our privacy policy.

Not the answer you're looking for? Browse other questions tagged or ask your own question.